Spin Up mit unbegrenzter Helizität

Stellen Sie sich vor, wir untersuchen die Spinquantisierung entlang der gleichen Achse wie der Impuls. Was ist, wenn ich einen Dirac-Spinor mit einem Spin-up, aber ohne eindeutige Helizität habe ( ψ L , ψ R 0 ):

u ( P ) = ( P · σ ξ P · σ ¯ ξ ) = ( ψ L ψ R )

Wie soll ich das verstehen? Wenn die Drehung entlang der P -Richtung positiv ist, sollte das Teilchen nicht eine eindeutige rechte Helizität und eine linke Komponente von 0 haben? Mit anderen Worten, sollten wir nicht haben u ( P ) ( 0 S Ö M e T H ich N G ) ?

Ja für ein masseloses Teilchen. Stellen Sie eine Frage zur teilweisen Überlappung von Helizität und Chiralität als Funktion der Masse?
Gar nicht. Ich frage auch nicht speziell nach masselosen Teilchen. Wenn die Drehung entlang der P -Richtung positiv ist, sollte das Teilchen nicht eine eindeutige rechte Helizität und eine linke Komponente von 0 haben? Dh sollte nicht u ( P ) ( 0 S Ö M e T H ich N G ) ?
Ja, auf Impuls ausgerichteter Spin bedeutet positive Helizität ... Ich bin mir nicht sicher, warum Sie Händigkeit ansprechen ....
Positive Helizität? Meinst du nicht rechtshändige Helizität? Es tut mir auch leid, dass ich meine Frage nicht klarstellen kann, aber ich bin mir nicht sicher, worauf ich näher eingehen soll?
Rechts. Nun, ein rechtshändiges (bzw. linkshändiges) Teilchen bedeutet, dass es ein positiver (bzw. negativer) Eigenwert des Helizitätsoperators ist.
Dies hat wahrscheinlich damit zu tun, dass die Helizität nicht Lorentz-invariant ist; Sie können auf einen Frame verstärken, in dem der Impuls des Partikels in die entgegengesetzte Richtung zeigt. Chiralität ist jedoch Lorentz-invariant.
Das hat absolut nichts mit meiner Frage zu tun.

Antworten (3)

OK, ich vermute, ich verstehe Ihre Verwirrung ... Sie werden einfach von der unangemessenen schlampigen Sprache "Rechtshänder-Helizität" vorgetäuscht. Es gibt keine solche Sache.

Nehmen Sie der Einfachheit halber den Spin in die z- Richtung und den Impuls in die z- Richtung. P μ = ( M 2 + P 2 , 0 , 0 , P ) , und untersuchen Sie Ihren Spinor auf der Weyl-Basis (chiral), ψ L = 1 2 ( 1 γ 5 ) ψ = ( ICH 2 0 0 0 ) ψ , ψ R = 1 2 ( 1 + γ 5 ) ψ = ( 0 0 0 ICH 2 ) ψ ,

u ( P ) = ( P · σ ξ P · σ ¯ ξ ) = ( ψ L ψ R ) .

Jetzt P · σ ξ = E P   ξ , Und P · σ ¯ ξ = E + P   ξ , so dass

u ( P ) = ( E P   ξ E + P   ξ ) ,
natürlich sowohl links- als auch rechtschiral. Die obere Komponente verschwindet im Allgemeinen nicht für positive Helizität .

Untersuchen Sie 3 Grenzen. Für p = 0, E = m und

u ( 0 ) = M ( ξ ξ ) ,
völlig ausgeglichen - die Helizität ist undefiniert.

Für m = 0 werden die oberen beiden Komponenten herausprojiziert, und nur die unteren beiden (eigentlich nur die dritte) überleben (überleben),

u ( P ) = 2 P ( 0 ξ ) ,
also wirklich rechtshändig: der Ursprung der schlampigen Bezeichnung positiver Helizität als "R", da Helizität und Chiralität identisch sind.

Für P M ,

u ( P ) 2 P ( M / 2 P   ξ ξ ) ,
also gibt es auch ein links-chirales Stück, aber das rechts-chirale Stück ist stark unterdominant. Auch dies ist ein Spinor mit positiver Helizität, der nicht von seinem linken chiralen Teil getrennt ist.

Die umgekehrte Aussage findet sich in Wikipedia : Für massive Teilchen haben unterschiedliche Chiralitätszustände (z. B. wie sie in den schwachen Wechselwirkungsladungen auftreten) sowohl positive als auch negative Helizitätskomponenten in Verhältnissen proportional zur Masse des Teilchens .

Das Lehrbuch von Itzykson & Zuber, 2-2-1 , liefert eine brauchbare Definition des Helizitätsoperators; in unserem Fall,

H ^ = 1 2 ( σ 3 0 0 σ 3 ) ,
analog zu obigem: es liest einen Eigenwert von +1/2 sowohl für den oberen, ψ L , und die untere, ψ R , Komponente, wie es sollte.

Es sei denn, man war sicher vor der sprachlichen Falle, die es zu vermeiden galt, oder man war ein Trottel für paradoxe Aussagen, um das Publikum auf Trab zu halten, es ist eine ungünstige Idee, mit Chiralitätsbegriffen herumzuwerfen, um die Helizität zu quantifizieren ...

  • PS. Ein treffenderer Titel wäre „Helizität mit unbestimmter Chiralität“ gewesen!
Danke dir für deine Bemühungen. Es mag an meinem Mangel an Intelligenz liegen, aber ich glaube nicht, dass dies meine Frage überhaupt beantwortet. Einfach, wenn der Spin mit ausgerichtet ist P sollte der obere Spinor nicht drin sein u ( P ) Sei 0 ? Übrigens, ich denke, Sie könnten sich diese andere Frage ansehen, die ich auch gepostet habe physical.stackexchange.com/questions/371978/… , es ist irgendwie verwandt.
Nein. Nein. Nein. Ich zeige dir warum und wie. Bitte überprüfen Sie Ihre Chiralität und verwechseln Sie sie nicht mit Helictity.
Wieder mit Chiralität. Ich danke Ihnen sehr, dass Sie versucht haben, mir zu helfen, und ich möchte überhaupt nicht unhöflich klingen, aber Sie bestehen weiterhin auf einer Sache und weigern sich, zu verstehen, was ich sage. Es ist, als würdest du mir sagen: "Nein, nein, das ist nicht deine Frage, deine Frage ist das ". Nein, ich weiß, was ich fragen will.

Es gibt zwei Dinge, von denen ich glaube, dass Sie sie möglicherweise missverstehen. Zum einen enthält der Spinor nicht nur Informationen über den Spin, sondern auch Informationen über Energie und Impuls.

u ¯ γ μ u = 2 ( E , P )
Sie müssen alle 4 Komponenten des Spinors verwenden, um dies zu beschreiben.

Die andere Sache ist das Aufrufen der Komponenten eines Spinors ψ L Und ψ R nur sinnvoll für masselose Teilchen (oder Teilchen, die sich sehr schnell bewegen).

Wenn Sie in Ruhe sind, P = ( M , 0 , 0 , 0 ) und die 0-te Komponente von σ ist die Identitätsmatrix, also ist Ihr Spinor gerecht

u ( P R e S T F R A M e ) = ( M ξ M ξ )
Und beide ψ L = ψ R egal, ob es nach oben oder unten (oder irgendetwas dazwischen) zeigt. Wie Sie die Komponenten in Richtung des Spins ankurbeln, wird einer der ψ L oder ψ R wird kleiner und der andere größer. Im Grenzfall sehr großer, aber endlicher Impulse wird je nach Helizität eine dieser beiden Komponenten sehr klein. Dies ist die Grenze, in der die Namen ψ R Und ψ L Sinn ergeben. Aber die „falsche“ Komponente muss immer noch ungleich Null sein, um Ihnen Informationen über das Momentum zu geben.

Das Problem ist also, dass ich das nicht verstanden habe ψ L Und ψ R sind Chiralitätseigenzustände , keine Helizitätseigenzustände .

In der Hochenergiegrenze, in der Helizität und Chiralität im Grunde dasselbe sind, wird der Zustand tatsächlich zu einem Eigenzustand (ich habe nachgerechnet, aber es ist zu lang, um hier zu posten, es sei denn, jemand fragt danach).